Tara wants to prove that a second pair of corresponding angles from KJN and LJM are congruent.


Determine a second pair of corresponding angles from KJN and LJM that are congruent. Then explain how you know that the two angles are congruent

Answers

Answer 1

To determine a second pair of corresponding angles from KJN and LJM that are congruent, we can start by identifying the first pair of corresponding angles.

angle JKN is  harmonious to angle LJM. thus, we need to find another brace of corresponding angles that involve these same two angles.   One possibility is to look at the  perpendicular angles formed by the  crossroad of KJ and JM. Angle KJM is  perpendicular to angle NJL. therefore, angle KJM in KJN corresponds to angle NJL in LJM. thus, these two angles are  harmonious.  

We can prove that these two angles are  harmonious using the  perpendicular angles theorem, which states that  perpendicular angles are always  harmonious. Since KJ and JM  cross at point J, angles KJM and NJL are  perpendicular angles and must be  harmonious.   thus, we've shown that the alternate brace of corresponding angles from KJN and LJM that are  harmonious are angle KJM and angle NJL.

Learn more about triangles at

https://brainly.com/question/29193314

#SPJ4

Tara Wants To Prove That A Second Pair Of Corresponding Angles From KJN And LJM Are Congruent. Determine

Related Questions

Brainliest if correct!_A particle is projected vertically upwards from a fixed point O. The speed of projection is u m/s. The particle returns to O 4 seconds later. Find:

a) the value of u

b) the greatest height reached by the particle

c) the total time of which the particle is at a height greater than half its greatest height

Thank you so much!

Answers

The value of the velocity, u is 19.6 m/s.

The greatest height reached by the particle is 19.6 m.

The total time during which the particle is at a height greater than half its greatest height is 2.33 s.

What is the value of the velocity, u?

a) To find the value of the velocity, u, we can use the formula for the time of flight of a vertically projected particle:

t = 2u/g

Since the particle returns to the same point after 4 seconds, we have:

2t = 4

Substituting the value of t in the first equation, we get:

u = gt/2 = 9.8 x 2

u = 19.6 m/s

b) To find the greatest height reached by the particle, we can use the formula for the maximum height reached by a vertically projected particle:

h = u^2/2g

Substituting the value of u, we get:

h = 19.6^2/(2 x 9.8)

h = 19.6 m

c) To find the total time during which the particle is at a height greater than half its greatest height, we can first find the height at which the particle is at half its greatest height:

h/2 = (u^2/2g)/2 = u^2/4g

Substituting the value of u, we get:

h/2 = 19.6^2/(4 x 9.8) = 24.01 m

So, the particle is at a height greater than half its greatest height when it is above 24.01 m.

Next, we can find the time taken by the particle to reach this height:

h = ut - (1/2)gt^2

24.01 = 19.6t - (1/2)9.8t^2

Solving this quadratic equation, we get:

t = 2.33 s or t = 4.10 s

The particle takes 2.33 s to reach a height of 24.01 m, and it takes another 1.67 s (4 - 2.33) to return to the ground.

Learn more about velocity at: https://brainly.com/question/80295?source=archive

#SPJ1

In a city of 72,500 people, a simple random sample of four households is selected from the 25,000 households in the population to estimate the average cost on food per household for a week. the first household in the sample had 4 people and spent a total of $150 in food that week. the second household had 2 people and spent $100. the third, with 4 people, spent $200. the fourth, with 3 people, spent $140.

required:
identify the sampling units, the variable of interest, and any auxiliary info mation associated with the units.

Answers

In this scenario, the sampling units are four households, the variable of interest is the average food cost, and auxiliary information associated with the units is the number of people in each household and total food cost.

Sampling Units: The sampling units are the four households selected from the 25,000 households in the population.

They are as follows:

1. Household with 4 people that spent $150 on food

2. Household with 2 people that spent $100 on food

3. Household with 4 people that spent $200 on food

4. Household with 3 people that spent $140 on food

Variable of Interest: The variable of interest is the average cost on food per household for a week.

Auxiliary Information: The auxiliary information associated with the units includes the number of people in each household and the total amount spent on food for that week.

To estimate the average cost on food per household for a week, follow these steps:

1. Calculate the total cost on food for all four households: $150 + $100 + $200 + $140 = $590

2. Divide the total cost by the number of households: $590 / 4 = $147.50

So, the estimated average cost on food per household for a week is $147.50.

Learn more about Variable of interest:

https://brainly.com/question/14356285

#SPJ11

Dante has a tent shaped like a triangular prism. The tent has an equilateral base that measures 5 feet on each side. The tent is 8 feet long and 4. 3 feet tall.


101. 5ft


120 ft.


141. 5 ft.


184 ft



What is the surface area of the tent

Answers

The surface area of Dante's tent is approximately 180 square feet.

To find the surface area of Dante's tent, we need to find the area of each of its faces and then add them up. Since the tent is in the shape of a triangular prism, it has three rectangular faces and two triangular faces.

First, let's find the area of each rectangular face. We know that the tent is 8 feet long and 4.3 feet tall. To find the width of each face, we need to use the Pythagorean theorem since the base of the tent is equilateral. So, we can use a triangle with sides of 5 feet (the base of the tent) and 4.3 feet (the height of the triangular face) to find the width of each rectangular face.

a^2 + b^2 = c^2
5^2 + 4.3^2 = c^2
25 + 18.49 = c^2
43.49 = c^2
c = √43.49
c ≈ 6.6 feet

Now that we know the width of each rectangular face, we can find the area of each face:

Area of rectangular face = length x width
Area of rectangular face = 8 x 6.6
Area of rectangular face ≈ 52.8 square feet

Next, let's find the area of each triangular face. Since the base of the tent is equilateral, each triangular face has a base of 5 feet and a height of 4.3 feet. The area of each triangular face is:

Area of triangular face = (base x height) / 2
Area of triangular face = (5 x 4.3) / 2
Area of triangular face ≈ 10.8 square feet

Now we can add up the areas of all five faces to find the total surface area of the tent:

Surface area of tent = 3 x area of rectangular face + 2 x area of triangular face
Surface area of tent = 3 x 52.8 + 2 x 10.8
Surface area of tent = 158.4 + 21.6
Surface area of tent ≈ 180 square feet

Therefore, the surface area of Dante's tent is approximately 180 square feet.

To know more about surface area, visit:

https://brainly.com/question/29298005#

#SPJ11

What three-dimensional figure is formed when the triangle shown is rotated around the dashed line?
A. cone
B. cylinder
C. double cone
D. hemisphere

Answers

Answer: C

Step-by-step explanation: after rotating, if you split it in half horizontally, you have two cones

The three-dimensional figure formed when the triangle is rotated around the dashed line through B and C is a cone.

What is a cone?

A cone is a three-dimensional geometric form with a flat base and a smooth, tapering apex or vertex. A cone is made up of a collection of line segments, half-lines, or lines that link the base's points to the apex, which is a common point on a plane that does not include the base.

When we rotate a two-dimensional shape around an axis, we create a three-dimensional solid. This process is known as "revolution" or "rotational symmetry".

In this particular case, we have a triangle that can be rotated around the line segment that connects points B and C. If we were to rotate the triangle around this axis, we would create a three-dimensional solid. To figure out what kind of solid this is, we can think about the cross-sections that would be created if we were to slice through the solid perpendicular to the axis of rotation.

If we were to slice through the solid perpendicular to the axis of rotation, we would get a circle. This means that the solid created by rotating the triangle is a cylinder.

To learn more about the cone;

brainly.com/question/16394302

#SPJ2

1 1/5, 1 2/5,1 2/5, 1 2/5, 1 3/5, 1 3/5 , 1 3/5, 1 4/5, 1 4/5, 2 What fraction of the packages weighed more than 1 2 /5 pounds?

Answers

2/5 of the packages weighed more than 1 2/5 pounds.

To find the fraction of packages that weighed more than 1 2/5 pounds, we need to count the number of packages that weighed more than 1 2/5 pounds and divide it by the total number of packages.

There are a total of 10 packages. Out of these, we can see that 4 packages weighed more than 1 2/5 pounds: 1 1/5, 1 3/5, 1 3/5, and 1 4/5.

Therefore, the fraction of packages that weighed more than 1 2/5 pounds is:

4/10

This can be simplified to:

2/5

Hence, only 2/5 of the package weighed more.

To learn more about weighed here:

https://brainly.com/question/23665105

#SPJ1

Which expression is equivalent to 1/4(8 - 6x + 12)?

Answers

The expression that is equivalent to 1/4(8 - 6x + 12) is 2 - 3x/2 + 6

What are algebraic expressions?

Algebraic expressions are simply defined as those mathematical expressions that are composed of terms, variables, their coefficients, their factors and constants.

These mathematical expressions are also comprised of arithmetic operations.

These operations are listed thus;

BracketParenthesesAdditionSubtractionMultiplicationDivision

From the information given, we have that;

1/4(8 - 6x + 12)

expand the bracket, we have;

8 - 6x + 12/4

Divide in group, we have;

8/4 - 6x/4 + 12/4

Divide the values

2 - 3x/2 + 6

Learn about algebraic expressions at: https://brainly.com/question/4344214

#SPJ1

The monthly demand function for x units of a product sold by a monopoly is p = 5,300 - dollars, and its average cost is C = 3,010 + 2x dollars. Production is limited to 100 units. Find the revenue function!

Answers

To find the revenue function, we need to multiply the price (p) by the quantity sold (x). The price function given is p = 5,300 - dollars, so we can substitute this into our revenue function as follows:

Revenue = p * x
Revenue = (5,300 - dollars) * x

We also know that production is limited to 100 units, so we need to take that into account when determining the revenue function. If x is greater than 100, then the revenue will be limited to 100 units sold. If x is less than or equal to 100, then the revenue will be based on the actual quantity sold.

To incorporate this constraint into our revenue function, we can use a piecewise function:

Revenue = { (5,300 - dollars) * 100           if x > 100
             (5,300 - dollars) * x             if x <= 100 }

Simplifying the piecewise function, we get:

Revenue = { 530,000 - (dollars * 100)        if x > 100
             (5,300 - dollars) * x             if x <= 100 }

Therefore, the revenue function for this monopoly is:

Revenue = { 530,000 - 100 * dollars        if x > 100
             (5,300 - dollars) * x             if x <= 100 }
Hi! To find the revenue function, we first need to determine the total revenue, which is the product of the price per unit (p) and the number of units sold (x). Given the demand function p = 5,300 - x dollars and the average cost function C = 3,010 + 2x dollars, we can find the revenue function as follows:

Revenue function, R(x) = p * x
R(x) = (5,300 - x) * x

By simplifying the equation, we get:
R(x) = 5,300x - x^2

So, the revenue function for this monopoly is R(x) = 5,300x - x^2.

To learn more about function visit;

brainly.com/question/12431044

#SPJ11

In parallelogram best, diagonals bs and et bisect each other at o.
1. if es = 10cm, how long is bt?
2. if be = 13cm, how long is ts?
3. if eo = 6cm and so = 7cm, what is the length of et? bs?
4. if et + bs = 18cm and so = 5cm, find et and bs.

Answers

When the parallelogram, diagonals bs and et bisect at each other at o, we get the following answers:

1. In a parallelogram, the diagonals bisect each other. So, if ES = 10 cm, then EO = OS = 5 cm. Since EO and OS are half of the diagonal ET, then ET = EO + OS = 5 cm + 5 cm = 10 cm. Similarly, diagonal BT will also be equal to 10 cm, as it has the same length as diagonal ET.

2. In a parallelogram, opposite sides are equal. So, if BE = 13 cm, then TS = 13 cm, as they are opposite sides.

3. If EO = 6 cm and SO = 7 cm, then the length of diagonal ET is EO + OS = 6 cm + 7 cm = 13 cm. Since the diagonals of a parallelogram are equal, the length of diagonal BS will also be 13 cm.

4. If ET + BS = 18 cm and SO = 5 cm, we can use the fact that diagonals bisect each other to find ET and BS. Let EO = x cm. Then, ET = 2x cm and BS = 2(5-x) cm. Now, we can set up the equation: 2x + 2(5-x) = 18. Solving for x, we get x = 4 cm. So, ET = 2x = 8 cm and BS = 2(5-x) = 10 cm.

To learn more about diagonal, refer below:

https://brainly.com/question/12274248

#SPJ11

1. The cost of renting a car for a day is $0.50 per mile plus a $15 flat fee.
(a) Write an equation to represent this relationship. Let x be the number of miles driven and y be the total cost for the day.
(b) What does the graph of this equation form on a coordinate plane? Explain.
(c) What is the slope and the y-intercept of the graph of the relationship? Explain

Answers

Answer:

a) y=0.50x+15  

b) The graph of this equation form on a coordinate plane is a line.

c) Slope  =0.50 and y-intercept = 15

Step-by-step explanation:

Let x = Number of miles driven by car

Given: The cost of renting a car for a day is $0.50 per mile plus a $15 flat fee.

a) Total cost = 0.50x+15

If y =total cost of renting the car, then y=0.50x+15   (i)

b) Above equation is similar to y= mx+c  (ii) [m = slope , xc=y-intercept] which a linear equation .

So the graph of this equation form on a coordinate plane is a line.

c) Comparing (i) and (ii)

m=0.50 , c=15

Hope this helps :)

HELPPPPPPP PLEASEEEE

Answers

Answer:

The first box and whisker plot

Step-by-step explanation:

A box and whisker plot gives you the five number summary for a set of data.  The five number summary is

The minimum/lowest value (looks like the top of capital T turned sideways and is the leftmost part of the box-and-whisker plot The first quartile or Q1, representing 25% of the data (the first point represented in the "box" of the plot and serves as an endpoint of the box)The median or Q2, representing 50%/the middle of the data (the line that splits the box into two parts/the line in the middle of the box)The third quartile or Q3, representing 75% of the data (the last point represented in the "box" of the plot and serves as another endpoint of the box)The maximum/highest value (also looks like the top of capital T turned sideways and is the rightmost part of the box-and-whisker plot

Maximum and minimum:

We know from the data that the minimum value is 100 and the maximum value is 200.  However, because both boxes available as answer choices have the correct minimum and maximum, we'll need to find more data.

Median:

We can start finding the median first by arranging the data from the least to greatest.  Then, we find the middle of the data.  Because there are 9 points and 9 is odd, we know that there will be 4 points to the left of the median and 4 points to the right of the median:

100, 100, 120, 120, 150, 165, 180, 180, 200

150 has 4 numbers both on its left and right sides so its the median.

Because both of the plots available as answer choices have the correct median, we we'll need to find more data.

First Quartile/Q1:

In order to find Q1, we must find the middle number of the four numbers to the left of the median.

Because we have an even number of points, we will get two middle numbers, 100 and 120.  To find the middle of all four points, we average these two numbers:

(100 + 120) / 2 = 220 / 2 = 110

Only the first box has the accurate Q1 value, so it's our answer.

We don't have to find Q3, since both boxes have the correct Q3, but only the first box has the correct minimum, correct Q1, correct median, correct Q3, correct maximum.

Can someone please help me ASAP? It’s due tomorrow.

Answers

Answer:

There are 16 total outcomes for tossing 4 quarters

This is because each coin flip has 2 possibilities, so if you flip the coin 4 times it will equal

2x2x2x2.

Every morning Jim runs for 15 minutes. If Jim runs 4 miles per​ hour, how far does Jim ​travel? Use the equation d=​rt, where d is​ distance, r is​ rate, and t is time.

Answers

We can use the formula d = rt, where d is distance, r is rate, and t is time to find the distance that Jim travels.

Given that Jim runs for 15 minutes, which is 1/4 of an hour. Also, his rate is 4 miles per hour.

Therefore, we can write:

d = rt

d = 4 miles/hour * (1/4) hour

d = 1 mile

So, Jim runs 1 mile every morning.

Dustin and Lucas decide to investigate Elle’s claims about the pudding. They obtain a sample of 200 chocolate pudding packs and find that only 72 of them contain more than 3. 25oz of pudding. A. (1 point) Construct a 92% CI for the overall proportion of pudding packs containing less than 3. 25oz of pudding. Make a conclusion at α = 0. 08 about whether these pudding packs truly are normally distributed with a mean of 3. 25oz. Use your confidence interval to justify your claim. B. (1 point) Construct a 99% CI for the overall proportion of pudding packs containing more than 3. 25oz of pudding. Make a conclusion at α = 0. 01 about whether these pudding packs truly are normally distributed with a mean of 3. 25oz. Use your confidence interval to justify your claim

Answers

(a) The 92% CI for overall proportion of pudding packs containing less than 3.25oz of pudding at α = 0.08 is (0.5806, 0.6994);

(b) The 99% CI for overall proportion of pudding packs containing more than 3.25oz of pudding at α = 0.01 is (0.2726, 0.4474).

Part (a) : To construct a confidence-interval for the overall proportion of pudding packs containing less than 3.25oz of pudding, we use the following formula : CI = p' ± [tex]z_{\frac{\alpha}{2} }[/tex] × √(p'(1-p')/n);

where p' is = sample proportion, [tex]z_{\frac{\alpha}{2} }[/tex]  is = critical z-value for the desired confidence level, and n is = sample size.

In this case, p' = (200-72)/200 = 128/200 = 0.64, the sample size is n = 200, and

We know that the critical z-value for a 92% confidence interval(α = 0.08) is approximately 1.75;

Substituting the values,

We get,

CI = 0.64 ± 1.75 × √(0.64(1 - 0.64)/200)

CI = 0.64 ± 0.059421;

CI = (0.5806, 0.6994);

Therefore, the required confidence interval is (0.5806, 0.6994).

Part (b) : In this case, p' = 72/200 = 0.36, the sample-size is n = 200, and

We know that the critical z-value for a 99% confidence interval (α = 0.01) is approximately 2.57;

Substituting the values,

We get,

CI = 0.36 ± 2..57 × √(0.36(1 - 0.36)/200)

CI = 0.36 ± 0.087426;

CI = (0.2726, 0.4474);

Therefore, the required confidence interval is (0.2726, 0.4474).

Learn more about Confidence Interval here

https://brainly.com/question/24131141

#SPJ4

2. Reemplaza los valores correspondientes de "a", "b" y "c", y calcula: a = -2 b = 3 c = 4 a) a + b – c = b) a – b + c = c) a + 2. B – 2c = d) (7. B) : (b + c) = e) a ∙ c + 2. B – 2. C = f) c · (b – a) =

Answers

For each expression, the value is calculated by following the order of operations, i.e. first solving any multiplication or division, then addition or subtraction. The resulting values are: a + b - c = -3, a - b + c = -1, a + 2b - 2c = -5, (7b)/(b+c) = 3, ac + 2b - 2c = -10, and c(b-a) = 20.

To calculate a + b - c, we substitute a = -2, b = 3, and c = 4. So,

a + b - c = -2 + 3 - 4 = -3

To calculate a - b + c, we substitute a = -2, b = 3, and c = 4. So,

a - b + c = -2 - 3 + 4 = -1

To calculate a + 2b - 2c, we substitute a = -2, b = 3, and c = 4. So,

a + 2b - 2c = -2 + 2(3) - 2(4) = -5

To calculate (7b) / (b + c), we substitute b = 3 and c = 4. So,

(7b) / (b + c) = (7(3)) / (3 + 4) = 21 / 7 = 3

To calculate ac + 2b - 2c, we substitute a = -2, b = 3, and c = 4. So,

ac + 2b - 2c = (-2)(4) + 2(3) - 2(4) = -10

To calculate c(b - a), we substitute a = -2, b = 3, and c = 4. So,

c(b - a) = 4(3 - (-2)) = 4(5) = 20

To know more about Substitution:

https://brainly.com/question/30284922

#SPJ4

Robert, by 3/4 pound of Grace, and divided into six equal portions. What is the way of each portion

Answers

Each portion of Grace weighs 1/8 pound.

What is weight?

It gauges how much gravity is pulling on a body.

If Robert has 3/4 pound of Grace and he wants to divide it into six equal portions, we can find the weight of each portion by dividing 3/4 by 6:

(3/4) / 6 = (3/4) * (1/6) = 1/8

So each portion of Grace weighs 1/8 pound.

Learn more about division on:

https://brainly.com/question/25289437

#SPJ4

this is reading btw and you get 23 points Fast pls

Think about the article you just read. Write two to three sentences describing what you would visualize in your mental model to understand how the two animals look different from each other.

Answers

The article red was titled "sense of emotion of dog and cat to humans"

To visualize the differences in feeling between Dogs and cats towards people, I would think of a dog swaying its tail and hopping up with fervor upon seeing its owner, whereas a cat may approach its owner more calmly and gradually with a loose tail.

I might picture the puppy gasping and looking for physical fondness, whereas the cat may lean toward to be petted or rubbed under the chin.

What is the  mental model?

Dogs show enthusiasm and affection towards owners, while cats exhibit different behaviors. Dogs show excitement through body language like wagging tails, jumping, seeking affection, and vocalizing.

Pets express joy and eagerness to be around humans, with cats being more reserved towards humans. Although affectionate, cats express emotions subtly such as calm body posture and soft chirping.

Learn more about  mental model from

https://brainly.com/question/31237269

#SPJ1

you select a marble without looking and then put it back if you do this 90 times what is the best predictions that you will pick an orange or a pink marble?

Answers

If you select a marble without looking and then put it back if you do this 90 times, the probability of selecting an orange or a pink marble on any given trial is 1/2 or 0.5.

If you select a marble without looking and then put it back, the probability of selecting any particular marble on any given trial is the same for all marbles. Assuming there are only two possible outcomes - selecting an orange marble or a pink marble - the probability of selecting an orange or a pink marble on any given trial is 1/2 or 0.5.

Since the probability of selecting an orange or a pink marble on each trial is the same, the best prediction for the number of times you will select an orange or a pink marble out of 90 trials is an equal number of times for each color. Therefore, the best prediction for the number of times you will select an orange or a pink marble is 45 times each.

This is only a prediction based on probability, and the actual number of times you select an orange or a pink marble may differ from the prediction due to chance. However, over a large number of trials, the actual outcomes should approach the predicted probabilities.

To learn more about predictions click on,

https://brainly.com/question/16801354

#SPJ1

If the points a,b and c have the coordinates a(5,2) , b(2,-3) and c(-8,3) show that the triangle abc is a right angled triangle

Answers

Points a,b and c satisfied the Pythagoras theorem. Thus, the triangle abc is a right angled triangle.

Define about the right angled triangle:

Every triangle has inner angles that add up to 180 degrees. A right angle and a right triangle are both formed when one of their internal angles is 90 degrees.

The internal 90° angle of right triangles is denoted by a little square in the vertex. The complimentary angles of the other two sides of a right triangle sum up to 90 degrees.The triangle's legs, which are typically denoted by the letters a and b, are the sides that face the complimentary angles.

Given coordinates :

a(5,2) , b(2,-3) and c(-8,3).

Find the distance between the points using the distance formula:

d = √[(x2 - x1)² + (y2 - y1)²]

ab = √[(2 - 5)² + (- 3 - 2)²]

ab = √[(-3)² + (- 5)²]

ab = √[9 + 25]

ab = √34

ab² = 34

bc = √[(2 + 8)² + (- 3 - 3)²]

bc  = √[(10)² + (- 6)²]

bc  = √[100 + 36]

bc  = √136

bc²  = 136

ac = √[(-8 - 5)² + (3 - 2)²]

ac = √[(-13)² + (1)²]

ac = √[169 + 1]

ac = √170

ac² = 170

Now,

(ac)² = (bc)² + (ab)²

170 = 136 + 24

170 = 170

This, points a,b and c satisfied the Pythagoras theorem. Thus, the triangle abc is a right angled triangle.

know more about the right angled triangle

https://brainly.com/question/64787

#SPJ1

• use the regression function from the previous step as a mathematical model for the demand function
(e.g. d(p)) and find the general expression for the elasticity of demand:
ep)

Answers

To find the general expression for the elasticity of demand (e_p), we need to differentiate the demand function with respect to price (p) and multiply it by the ratio of price to quantity (p/q). The elasticity of demand measures the responsiveness of quantity demanded to changes in price.

The general expression for elasticity of demand (e_p) can be calculated as:

e_p = (dQ/dp) * (p/Q)

Where dQ/dp represents the derivative of the demand function with respect to price, and Q represents the quantity demanded.

The elasticity of demand helps us understand how sensitive the quantity demanded is to changes in price. If e_p is greater than 1, demand is considered elastic, meaning that quantity demanded is highly responsive to price changes. If e_p is less than 1, demand is inelastic, indicating that quantity demanded is less responsive to price changes.

In conclusion, the general expression for the elasticity of demand (e_p) is calculated by taking the derivative of the demand function with respect to price and multiplying it by the ratio of price to quantity. This measure helps determine the responsiveness of quantity demanded to changes in price.

To know more about elasticity of demand refer here:

https://brainly.com/question/30704413

#SPJ11

Solve the differential equation. dy + 6ydx=9e -⁶x dx y=

Answers

The general solution to the given differential equation:

[tex]y = (9x + C)e^(-6x)[/tex]

To solve the given differential equation, dy + 6ydx = 9e^(-6x)dx, we can first rewrite it as a first-order linear differential equation:

[tex]dy/dx + 6y = 9e^(-6x)[/tex]

Now, we will find the integrating factor, which is e^(∫P(x)dx), where P(x) is the coefficient of y in the equation:

Integrating factor =[tex]e^(∫6dx) = e^(6x)[/tex]

Next, we multiply the entire differential equation by the integrating factor:

[tex]e^(6x)(dy/dx + 6y) = 9e^(-6x)e^(6x)[/tex]

This simplifies to:

[tex]e^(6x)(dy/dx) + 6e^(6x)y = 9[/tex]

Now, the left side of the equation is the derivative of the product of y and the integrating factor:
[tex]d/dx(y * e^(6x)) = 9[/tex]

To solve for y, integrate both sides with respect to x:

[tex]∫[d/dx(y * e^(6x))]dx = ∫9dx[/tex]

This results in:

[tex]y * e^(6x) = 9x + C[/tex]

Finally, we solve for y by dividing by e^(6x):

[tex]y = (9x + C)e^(-6x)[/tex]

And that is the general solution to the given differential equation.

To learn more about differential equation, refer below:

https://brainly.com/question/14620493

#SPJ11

square peg sydney smith wrote in ""on the conduct of the understanding"" that it is im-possible to fit a square peg in a round hole.

Answers

On the basic of square peg sydney smith written point the probability that it is im-possible to fit a square peg in a round hole is equals to the zero.

Probability is defined as the number of chances of occurrence of an event. It is the ratio of the number of favorable outcomes to the total outcomes in that sample space. Mathematical formula is written as, probability of an event, P(E) = (Number of favorable outcomes)/(Total possible outcomes). Now, we have specify that according to sydney smith written in on the conduct of the understanding about square peg that it is impossible to fit a square peg in a round hole. Let consider an event A of fit a square peg in a round hole. We have specify that it is impossible to fit a square peg in a round hole. So, the favourable possible outcomes for event A = 0. Therefore, the probability that to fit a square peg in a round hole, P(A) = 0

Hence, required probability value is zero.

For more information about probability, visit:

https://brainly.com/question/25870256

#SPJ4

Find the surface area of the composite figure.
2 in.
4 in.
9 in.
SA
=
7 in.
2
[?] in.²
4 in.
4 in.
If you'd like,
you can use a
calculator.

Answers

Answer:

  236 in²

Step-by-step explanation:

You want the surface area of the figure comprised of two cuboids.

Area

The surface area of the figure will be the sum of the total surface area of the purple cuboid, plus the lateral surface area of the yellow cuboid.

  SA = 2(LW +H(L +W)) + Ph

  SA = 2(9·4 +2(9 +4)) +(4·4)(7) = 236 . . . . square inches

The surface area of the composite figure is 236 square inches.

__

Additional comment

You can consider the face on the right side to be equal in area to the area of the purple cuboid that is covered by the yellow one. So, figuring the total area of the purple cuboid effectively includes the area of the face on the right side.

Then the remaining part of the area of the yellow cuboid is the area of the four 7×4 rectangles that are its lateral area.

<95141404393>

Gareth pays $60 for 9m of climbing rope. How much will Sophie pay for 15m at the same store?

Answers

Sophie will pay money equivalent to $100 for 15m at the same store.

What is Money?

The term "money" in mathematics refers to a form of payment, such as bills, coins, and demand deposits, that is used to purchase goods and services. Money is used to pay for the worth or price of an item or service.

A country's monetary system is referred to as its currency.

In the case of Gareth,

Money paid for 9 m of climbing [tex]=\$60[/tex]

Money paid per m of climbing [tex]=\$60\div9[/tex]

Thus, money paid by Sophie for 15 m of climbing [tex]= 15 \times (60\div9)[/tex]

[tex]\boxed{\bold{= \$100}}[/tex]

Hence Sophie will pay money equivalent to $100 for 15m at the same store.

Learn more about money here:

https://brainly.com/question/24373500

Answer:

$100

Step-by-step explanation:

We know that for 9m of rope, Gareth had to pay $60.

The question is asking us to find out how much Sophie will pay for 15m of rope.  To do this, we have to find out how much is paid per meter of rope.

[tex]60/9\\=6\frac{2}{3}[/tex]

For the sake of not using fractions, let's keep it as an improper fraction: 60/9

So, we can write an equation for the price of 15m of rope:

(60/9)·15

=100

So, Sophie will pay $100 for 15m of rope.

Hope this helps! :)

Geometry
three squares with areas of 64, 225, and 289 square units are arranged so that when their vertices coincide a triangle is formed. find the area of that triangle.

please explain how you solved this along with the answer.

Answers

Answer:

The area of the largest square is 289 square units, because it is the sum of areas of the two smaller squares, 64 square units and 225 square units.

Step-by-step explanation: JUST PASSED IT ON STUDY ISLAND 100% CORRECT ANSWER

Please help it due soon and the answer is meant to be in kg

Answers

Answer: 20 kg

Step-by-step explanation:

You follow the line of best fit until 50cm

Then you trace across and look at the x-axis.

There you will find that the dog will be 20kg at 50cm using the line of best fit.

The profit in dollars from the sale of x expensive watches is P(x) = 0.08x² - 5x + 6x0.2 - 5200 Find the marginal profit when (a) x = 300. (b) x = 2000, (c) X = 5000, and (d) x = 12,000.

Answers

The marginal profit in dollars for the sale of expensive watches when approximately $1912.61.

Find the marginal profit in dollars from the sale?

We need to find the marginal profit in dollars from the sale of x expensive watches for the given profit function P(x) = 0.08x² - 5x + 6x^0.2 - 5200 when (a) x = 300, (b) x = 2000, (c) x = 5000, and (d) x = 12,000.

Find the derivative of the profit function P(x), which represents the marginal profit.
P'(x) = dP(x)/dx = 0.16x - 5 + (6 * 0.2 * x^(-0.8))

Calculate the marginal profit for each specified value of x:

x = 300:
P'(300) = 0.16(300) - 5 + (6 * 0.2 * 300^(-0.8)) ≈ 42.57

x = 2000:
P'(2000) = 0.16(2000) - 5 + (6 * 0.2 * 2000^(-0.8)) ≈ 317.52

x = 5000:
P'(5000) = 0.16(5000) - 5 + (6 * 0.2 * 5000^(-0.8)) ≈ 794.57

x = 12,000:
P'(12,000) = 0.16(12,000) - 5 + (6 * 0.2 * 12,000^(-0.8)) ≈ 1912.61

So, the marginal profit in dollars for the sale of expensive watches when (a) x = 300 is approximately $42.57, (b) x = 2000 is approximately $317.52, (c) x = 5000 is approximately $794.57, and (d) x = 12,000 is approximately $1912.61.

Learn more about derivative.

brainly.com/question/25324584

#SPJ11

SIMPLIFYYY THIS EXPRESSION

Answers

Answer:

x - 5y

Step-by-step explanation:

6x - 8y - 5x + 3y =

= 6x - 5x - 8y + 3y

= x - 5y

Answer:

1x + 11y

Step-by-step explanation:

Find the sum of the first 10 terms of the following series, to the nearest integer.


8,20/3,50/9

Answers

The sum of the first 10 terms of the given series  8,20/3,50/9... is 140.

Given series: 8,20/3,50/9...

The given series is not in a standard form, but it appears to be an arithmetic sequence with a common difference of  4/3. To check this, we can find the difference between consecutive terms:

20/3-8=4/3

50/9-20/3=4/3

Thus, the common difference is indeed [tex]\frac{4}{3}[/tex].

We notice that each term of the series can be written as:

a_n=a+(n-1)d

a_n=8+(n-1)(4/3)

where n is the index of the term, and 4/3  is the common difference between the consecutive terms.

To find the sum of the first 10 terms of the series, we use the formula for the sum of an arithmetic series:

S=(n/2)[2a_1+(n-1)d]

where S is the sum of the series, a_1 is the first term of the series, d is the common difference, and n is the number of terms to be added.

Substituting the given values, we get:

S=(10/2)[2*8+(10-1)(4/3)]

Simplifying the expression:

S=5[16+9(4/3)]

S=5[16+12]=5(28)=140

Therefore, the sum of the first 10 terms of the series is 140.

To know more about sum refer here:

https://brainly.com/question/31335425#

#SPJ11

Maria invests $6,154 in a savings account with a fixed annual interest rate of 8% compounded weekly. What will the account balance be after 10 years? There are 52 weeks in a year. (Round our answer to the nearest cent)

Answers

Answer:

A = P(1 + r/n)^(n*t) is the formula

Where:

A = the account balance after t years

P = the principal amount (initial investment)

r = the annual interest rate (as a decimal)

n = the number of times the interest is compounded per year

t = the time in years

P = $6,154

r = 0.08 (8% expressed as a decimal)

n = 52 (compounded weekly)

t = 10

A = 6154(1 + 0.08/52)^(52*10)

A ≈ $14,239.44

Therefore, the account balance after 10 years will be approximately $14,239.44.

Segments OT and OV are?

Answers

True, The more variables that are deployed to segment a market, the more useful is the resulting segmentation likely to be.

We have,

The factors which are be used to member a request are the segmentation variables. Common variables include demographic, geographic, psychographics and behavioral considerations.

Quantifiable population characteristics, similar as age, gender, income, education, family situation.

The primary ideal of segmentation is to identify guests with analogous attributes, and to find which parts of guests that are seductive from a profit perspective.

Understanding the request segmentation allows marketers to produce a more effective and effective marketing blend.

Hence, True, The more variables that are deployed to segment a market, the more useful is the resulting segmentation likely to be.

To know more about segment a market , visit: brainly.com/question/27993208

#SPJ1

complete question:

The more variables that are deployed to segment a market, the more useful is the resulting segmentation likely ot be.[See p.104]Group of answer choicesTrueFalse

Other Questions
I dont know how to do this question if anyone does please put the steps thank you. As fluids are transported over a long distance, what happens to the fluid pressure in the pipes? Why does this happen? Is this correct for problem 15? What did uncle Henry and Dorothy hear from the far north In what way are Italian Fascism and German Nazism in the 1930s similar? A .A They both used a democratic form of government to make major decisions . B . Both were led by dictators who use nationalism and terror to maintain control over citizens. C. Both were communist governments that attacked neighboring countries to increase their empires D .Bother allowed the government to control industry and agriculture in order to make all citizens equal Help whats the answer? What volume of 7.8 M copper (II) sulfate stock solution is needed to prepare 3.25 L of a 5.4 M solution?WILL MARK BRAINLIEST What is the main reason plants grow fruit?Ato provide delicious food for humans and other animalsBto stop animals from spreading seedsCto encourage bees to pollinateDto keep seeds safe and make them easier to spread Each time you are honest and conduct yourself with honesty, a success force will dove you toward greater success. Each me you le, even with a little whit lie, there are strong forces pushing you toward failure Joseph Sugarman According to this quote, honesty is important to ?A.) self- esteem B.) success C.) no one can someone help with this please? there was a video attached to this.. (its not that long at all but ill out the link in the comments incase anyone may need it) you also have to look at the previous question i posted yesterday to answer/understand this someone answered it already btw but just look at what they said and look at this 5) assume that a typical lighting strike delivers -25 [c] to the earth, and the average voltage drop between the cloud and ground (voltage of cloud minus voltage of ground) is -75 [mv] during the time the charge is delivered. assume that a lightning strike hits the earth from the cloud every 10 [s], and that the thunderstorm lasts one hour. assume that somehow all of the energy in all of the lightning strikes could be captured. how long would this stored energy be able to supply a city, assuming that the supply rate is the same as that coming from a large power plant, rated at 1,000 [mw]? An isosceles triangle has an angle that measures 78. What measures are possible for the other two angles? 6. Acorn Squirrel Crow Coyote1,000 kcal are available for the 1st trophic level, 100kcal for the 2nd trophic level, 10 kcal for the 3rdtrophic level. How many kcal are available for thetertiary consumer?Fill in the organisms and energy amounts on thepyramid to the right PLEASE THINK ABOUT THIS!!Research John Wooden's Pyramid of Success- Who created it and why? What was it created for? Why are the bricks stacked this way? CHOOSE 5 to talk about in relation to your life. Is this something you have or is it something you need to work on? if a property owner has been using their property as a manufacturing facility for the past 40 years, but the current zoning laws only allow residential use in the neighborhood, the manufacturing facility will be considered as a/an...? How to master English language in writing and speaking? To find the standard deviation of the liquid measure of oil in barrels, the oil company measures 25 randomly selected barrels and find the standard deviation of the samples to be s=. 34. Find the 92% confidence interval for the population standard deviation I cannot figure out how to limit the number that they can make the bars of the turtle go too. I need to find a way to make them not go any higher than 200.import turtle Jane = turtle.Turtle() bar1 = int(input("What is the height of the first bar?" ))bar2 = int(input("What is the height of the second bar?" ))bar3 = int(input("What is the height of the third bar?" ))bar4 = int(input("What is the height of the fourth bar? "))Jane.left(90)def bar(height): Jane.forward(height) Jane.left(90) Jane.forward(20) Jane.left(90) Jane.forward(height) Jane.right(180)bar(bar4)bar(bar3)bar(bar2)bar(bar1) PLEASE HELP ME PLEASE I REALLY NEED HELP IM LOST What was the outcome/impact/appeal of Marvel Comics and the hulk?